Whats wrong with my work for improper integral?

Click For Summary
The discussion revolves around the evaluation of the improper integral ∫^{∞}_{1} (dx/(x(2x+5))). The original poster initially miscalculated the limits, particularly confusing the evaluation of natural logarithms. A key correction highlighted was that nl(1) should actually be nl(2) when evaluating nl(2x+5) at x = 1. Additionally, the issue of handling the expression ∞ - ∞ was pointed out, which contributed to the misunderstanding of the integral's limits. Ultimately, the poster acknowledged the mistake and expressed gratitude for the clarification.
DeadOriginal
Messages
274
Reaction score
2

Homework Statement


∫^{∞}_{1}\frac{dx}{x(2x+5)}

Homework Equations


Mathematica told me that the answer was \frac{1}{5}nl(\frac{7}{2})

The Attempt at a Solution


This is my work.

∫^{∞}_{1}\frac{dx}{x(2x+5)} = lim_{M\rightarrow∞}∫^{M}_{1}\frac{dx}{x(2x+5)} = lim_{M\rightarrow∞}∫^{M}_{1}\frac{\frac{1}{5}}{x}+\frac{\frac{-2}{5}}{2x+5} = lim_{M\rightarrow∞}((\frac{1}{5}∫^{M}_{1}\frac{1}{x})-(\frac{2}{5}∫^{M}_{1}\frac{1}{2x+5})) = lim_{M\rightarrow∞}\frac{1}{5}(nl(x)-nl(2x+5))^{M}_{1} = lim_{M\rightarrow∞}\frac{1}{5}((nl(M)-nl(2M+5))-(nl(1)-nl(7))) = lim_{M\rightarrow∞}\frac{1}{5}((nl(M)-nl(2M+5))-(0-nl(7))) = \frac{1}{5}((∞-∞)+nl(7)) = \frac{1}{5}nl(7) .

I'm looking at it over and over again but I just can't seem to figure out where that nl(2) from mathematica came from. Could someone please look at my work and tell me what is wrong with it? I'd greatly appreciate it.
 
Last edited:
Physics news on Phys.org
I've never seen natural log as nl. Usually it's either ln, or simply log.

Where you have nl(1), you should have nl(2) for nl(2x+5) evaluated at x = 1.

That should do it !
 
SammyS said:
I've never seen natural log as nl. Usually it's either ln, or simply log.

Where you have nl(1), you should have nl(2) for nl(2x+5) evaluated at x = 1.

That should do it !

Hi SammyS,

Thanks for the help. I knew nl looked kind of weird. haha. For the sake of keeping it from getting confusing I'll finish this thread with it.

I don't understand what you mean though. For nl(1), I had nl(x) and I evaluated at x = 1. That's why I had nl(1). Shouldn't nl(2x+5) evaluated a x = 1 be nl(7)?
 
You have a problem evaluating the upper limit of the integral as M approaches infinity. In your next to last expression, infinity - infinity is not equal to zero.

It would be easier to determine the indefinite integral before applying the limits of integration.
 
DeadOriginal said:
Hi SammyS,

Thanks for the help. I knew nl looked kind of weird. haha. For the sake of keeping it from getting confusing I'll finish this thread with it.

I don't understand what you mean though. For nl(1), I had nl(x) and I evaluated at x = 1. That's why I had nl(1). Shouldn't nl(2x+5) evaluated a x = 1 be nl(7)?
Yes, you are correct. I merely had a brain cramp. (DUH !)

Look at SteamKing's post. regarding ∞ - ∞. I think that's where you may find the ln(2) you're looking for.
 
OOOOOOOOOOHHHHHHHHHHHHHHHHHHH. *bangs head on table*

Thanks a lot.
 
Question: A clock's minute hand has length 4 and its hour hand has length 3. What is the distance between the tips at the moment when it is increasing most rapidly?(Putnam Exam Question) Answer: Making assumption that both the hands moves at constant angular velocities, the answer is ## \sqrt{7} .## But don't you think this assumption is somewhat doubtful and wrong?

Similar threads

  • · Replies 6 ·
Replies
6
Views
2K
  • · Replies 6 ·
Replies
6
Views
1K
  • · Replies 105 ·
4
Replies
105
Views
6K
  • · Replies 8 ·
Replies
8
Views
2K
  • · Replies 8 ·
Replies
8
Views
1K
  • · Replies 27 ·
Replies
27
Views
3K
Replies
4
Views
2K
  • · Replies 6 ·
Replies
6
Views
2K
  • · Replies 7 ·
Replies
7
Views
2K
  • · Replies 10 ·
Replies
10
Views
2K